Stetigkeit zeigen

Neue Frage »

xaittt Auf diesen Beitrag antworten »
Stetigkeit zeigen
Hi

Ich habe folgende Funktion gegeben:



Um stetigkeit zu zeigen, reicht es doch wenn ich zeige dass die Funktion in (0,0) stetig ist, da die Funktion ja ansonsten einfach nur eine Komposition stetiger Funktionen ist.
Stimmt das?



lg
IfindU Auf diesen Beitrag antworten »
RE: Stetigkeit zeigen
Stimmt.
xaittt Auf diesen Beitrag antworten »

Danke. Ich habe das mit dem Kriterium so gemacht:





Passt so oder? Eine Frage bzgl den Beträgen, kann man hier überhaupt den Betrag nehmen oder nimmt man hier die allgemeine Norm ?


Desweiteren sollte man noch zeigen dass die Funktion in (0,0) nicht diffbar ist. Das habe ich so gemacht, in dem ich zeige dass die partielle Ableitung nicht stetig in (0,0) ist.
Die partielle Ableitung nach x lautet:



Sei eine Nullfolge in . Dann gilt:

nicht stetig in

Passt das so?


lg
IfindU Auf diesen Beitrag antworten »

Wie ist denn die "allgemeine" Norm definiert?

Das erste ist im Prinzip richtig, allerdings nicht formal richtig aufgeschrieben.

Ansonsten kann eine Funktion differenzierbar sein, ohne stetig differenzierbar zu sein. Was du gemacht hast ist zu zeigen, dass sie in der 0 nicht stetig diff'bar ist -- sie könnte dennoch differenzierbar sein.
xaittt Auf diesen Beitrag antworten »

Zitat:
Original von IfindU
Wie ist denn die "allgemeine" Norm definiert?

Das erste ist im Prinzip richtig, allerdings nicht formal richtig aufgeschrieben.

Ansonsten kann eine Funktion differenzierbar sein, ohne stetig differenzierbar zu sein. Was du gemacht hast ist zu zeigen, dass sie in der 0 nicht stetig diff'bar ist -- sie könnte dennoch differenzierbar sein.

Oh ja da hab ich mich zu früh gefreut unglücklich

Gibt es denn eine andere Möglichkeit ausser das direkt über die Definition der totalen Diffbarkeit zu machen, denn wenn ich mir die Def. anschaue:



Wie finde ich dieses L? Bzw. wie zeige ich das dieses nicht existiert?


Zudem haben wir noch einen Satz in der VOrlesung aufgeschrieben der geht so:
Ist f in einem Punkt a total diffbar mit existiert, d.h. die Richtungsableitung in alle Richtungen.

Wenn ich den Satz verneine:
mit existiert nicht totale Ableitung in a existiert nicht.

Stimmt die Verneinung so? Das würde ja heissen ich bräuchte nur so ein x zu finden, in dessen Richtung keine Richtungsableitung existiert?


lg
xaittt Auf diesen Beitrag antworten »

OK habe noch eine Idee:

Angenommen die obige Funktion ist total diffbar in (0,0), dann existiert ja auch die Jacobi-Matrix. Das wäre in diesem Fall einfach nur der (0,0) Vektor, d.h. das L dass ich vorhin gesucht habe ist einfach 0. D.h. mein Differentenquotient schaut so aus



Soo laut Wolframalpha existiert der Grenzwert nicht, was ja schonmal gut wäre, nur ich weiss leidernicht wie ich das zeigen kann. Stimmt das überhaupt soweit?


lg
 
 
IfindU Auf diesen Beitrag antworten »

Ich war leider ein paar Tage verreist.

Wie kommst du darauf, dass die Ableitung (0, 0) sein muss? Wenigstens ich musste mir Gedanken dazu machen.

Aber nun gut, es stimmt, und damit ist der linke Ausdruck genau der zu untersuchende. Danach die Umformung ist dann extrem falsch, ich hoffe einmal ein Tippfehler.

Dann ist genau zu zeigen, dass dieser Grenzwert nicht existiert. Dazu wählt man dann passende Nullfolgen s.d. man für n gegen unendlich nicht bei 0 in dem Ausdruck landet.
Neue Frage »
Antworten »



Verwandte Themen

Die Beliebtesten »
Die Größten »
Die Neuesten »